\documentclass[compose]{exam-n} \begin{document} \begin{question}{20} \author{Graham Woan} \part The recently-launched \emph{Swift} Gamma Ray Burst telescope is expected to detect about 200 bursts of gamma rays during its 2-year lifespan. Explain why the Poisson distribution, \begin{equation*} P(n|\lambda)=\exp(-\lambda)\lambda^n /n! \end{equation*} is appropriate to describe the probability of detecting $n$ bursts, and carefully explain the significance of the parameter $\lambda$. Table~\ref{t:excess} has absolutely nothing to do with this question, and its presence here is proof positive of the existence of aliens who wish to do us typographical harm.\partmarks{4} % This table is here to check that the table in the _solution_ to the % previous question doesn't cause the table counter to be % incremented. This should be Table 2, whether or not solutions are % being shown. \begin{table} \begin{centering} \begin{tabular}{r|l} \hline left&right\\ \hline \end{tabular} \caption{\label{t:excess}This is a table} \end{centering} \end{table} Given the above, estimate the probability that \emph{Swift} will detect more than three bursts on any particular calendar day. % Fill up the line, so that we check that (default) \partmarks doesn't % include \@partmarksspace, since (default) \partmarks is now fully in % the margin. Blah. Blah. Blaah. Fill the line.\partmarks{6} \part Explain how Bayesian inference uses the observed number of bursts to infer the true burst rate at the sensitivity limit of \emph{Swift}, and explain the significance of the posterior probability distribution for $\lambda$. \partmarks{5} \begin{solution} \includepdf[pages=-]{numerical1-solution.pdf} \end{solution} Assuming that the posterior, $p$, for $\lambda$ can be approximated as a gaussian, show that, quite generally, the uncertainty in $\lambda$ inferred from \emph{Swift} will be \begin{equation*} \sigma \simeq \left( -\frac{\partial^2\ln p}{\partial \lambda^2}\Big|_{\lambda_0} \right)^{-1/2}, \end{equation*} where $\lambda_0$ is the most probable value of $\lambda$. \partmarks{5} \end{question} \end{document}